Đến nội dung

LoveMath1234567 nội dung

Có 15 mục bởi LoveMath1234567 (Tìm giới hạn từ 29-04-2020)


Sắp theo                Sắp xếp  

#719646 Phương pháp giải Phương trình

Đã gửi bởi LoveMath1234567 on 21-01-2019 - 15:43 trong Phương trình - hệ phương trình - bất phương trình

Phương pháp cô lập tham số.

pp bạn nói đa phần để xét tính đơn điệu của hàm số

Cho mình nói thêm :

 Sau khi đặt ẩn t thầy m vẽ bảng biến thiên của hàm số : y= t bình -4t

Để pt có 4 nghiệm thì m phải lớn hơn -4 nhỏ hơn -3 .....

Bạn có thể xem cho mình xem đây là pp j ko ...Mình cần hk nó gấp




#719634 Phương pháp giải Phương trình

Đã gửi bởi LoveMath1234567 on 20-01-2019 - 17:19 trong Phương trình - hệ phương trình - bất phương trình

Đề bài : Có bao nhiêu giá trị m để pt : $ x^{2}-4\sqrt{x^{2}+1}-\left ( m -1\right )= 0$ có 4 nghiệm phân biệt 

 P/S: Thầy mình giải theo cách : Đặt $ t= \sqrt{x^{2}+1}$

         Biến đổi pt ban đầu thành : $ t^{2}-4t= m$ rồi lập bảng xét dấu ,....

Giúp mình giải thích đây là phương pháp gì thế...Thanks nhìu




#719563 Cho $A(1;2)$, $B(3;4)$, tìm trên $Ox$ điểm...

Đã gửi bởi LoveMath1234567 on 18-01-2019 - 06:34 trong Phương pháp tọa độ trong mặt phẳng

a) Lấy A' đối xứng với A qua trục hoành. Đặt N là giao điểm của A'B với Ox

Với mọi M thuộc Ox ta có

MA + MB = MA' +MB $\geq A'B=\sqrt{(3-1)^{2}+(4+2)^{2}}=2\sqrt{10}$

Đẳng thức xảy ra <=> M$\equiv$N<=> M thuộc đoạn HK

và $\frac{MH}{MK}=\frac{A'H}{BK}=\frac{2}{4}=\frac{1}{2}$

<=> $\frac{\overline{MH}}{\overline{MK}}= -\frac{1}{2}$

<=> $x_{M}=\frac{x_{H}+\frac{1}{2}x_{K}}{1+\frac{1}{2}}=\frac{1+\frac{3}{2}}{\frac{3}{2}}=\frac{5}{3}$

<=> M(5/3;0)

HK ở đâu ra thế ạ




#719155 Trong các tập hợp sau, tập nào là tập con của tập nào ?

Đã gửi bởi LoveMath1234567 on 06-01-2019 - 21:12 trong Đại số

 

a) DBCADBCA

 

b) AB

 

sao b lại là con c thế bạn




#719153 Trong các tập hợp sau, tập nào là tập con của tập nào ?

Đã gửi bởi LoveMath1234567 on 06-01-2019 - 21:08 trong Đại số

a) $D\subset B\subset C\subset A$

b) $A\subset B$

thế nghĩa là hình chữ nhật là  hình thoi????.....




#718986 Tổng hợp các bài BĐT

Đã gửi bởi LoveMath1234567 on 02-01-2019 - 21:26 trong Bất đẳng thức và cực trị

 

Topic này chủ yếu là coppy nội dung đề bài và lời giải của các mem lại, các ĐHV THPT có thể coppy tiếp CD13 nhưng nên để chung trong một khung. Tính đúng sai của các lời giải thì kiểm tra lại sau.

 

Bắt đầu vậy!

 

Bài 1:

Cho abc=1 va $a^{3}> 36.  CMR  :\frac{a^{2}}{3}+b^{2}+c^{2}> ab +bc+ca$}

Lời giải:

$VT-VP=\frac{a^{2}}{4}+b^{2}+c^{2}-ab-bc+2bc+\frac{a^{2}}{12}=(\frac{a}{2}-b-c)^{2}+\frac{a^{2}-36bc}{12}>0\Rightarrow$ đpcm

Cách khác:

Từ giả thiết suy ra $a>0$ và $bc>0$. Bất đẳng thức cần chứng minh tương đương với
\[\dfrac{a^2}{3}+(b+c)^2-3bc-a(b+c)\ge 0\\ \iff \dfrac{1}{3}+\left(\dfrac{b+c}{a}\right)^2-\dfrac{b+c}{a}-\dfrac{3}{a^3}\ge 0\]
Vì $a^3>36$ nên \[\dfrac{1}{3}+\left(\dfrac{b+c}{a}\right)^2-\dfrac{b+c}{a}-\dfrac{3}{a^3}> \left(\dfrac{b+c}{a}\right)^2-\dfrac{b+c}{a}+ \dfrac{1}{4}= \left(\dfrac{b+c}{a}-\dfrac{1}{2}\right)^2 >0\]
 
 

 

Bài 2:

Với a,b,c >0; n ∈ N*.CMR:

$\frac{a^{n}}{b+c}+\frac{b^{n}}{a+c}+\frac{c^{n}}{a+b}\geq \frac{3}{2}\left ( \frac{a^{n}+b^{n}+c^{n}}{a+b+c} \right )$

Lời giải:

$\sum \frac{a^{n}}{b+c}\geq \frac{1}{3}(\sum a^{n})(\sum \frac{1}{a+b})\geq \frac{1}{3}(\sum a^{n})(\frac{9}{2(a+b+c)})=\frac{3}{2}(\frac{\sum a^{n}}{\sum a})$

 

 

 

Bài 3:

Cho $x,y,z >0$ thỏa điều kiện $x^{2}+y^{2}+z^{2}=9$

Tìm giá trị nhỏ nhất của $P=\frac{x^5}{y^2}+\frac{y^5}{z^2}+\frac{z^5}{x^2}$

Lời giải:

Theo $Cauchy$ Ta có:

$$\dfrac{x^5}{y^2}+\dfrac{x^5}{y^2}+\sqrt{3}y^2+\sqrt{3}y^2+3\sqrt{3}\ge \sqrt{3}x^2$$

Cách khác:

Sử dụng Cauchy-Schwarzt ta có 

       $\frac{x^5}{y^2}+\frac{y^5}{z^2}+\frac{z^5}{x^2}\geqslant \frac{(x^3+y^3+z^3)^2}{xy^2+yz^2+zx^2}$

Sử dụng Cauchy-Schwarzt và AM-GM ta có 

       $xy^2+yz^2+zx^2\leqslant \sqrt{(x^2+y^2+z^2)(x^2y^2+y^2z^2+z^2x^2)}\leqslant \sqrt{\frac{(x^2+y^2+z^2)^3}{3}}=3$

Do đó $P\geqslant \frac{(x^3+y^3+z^3)^2}{3}\geqslant \frac{(x^2+y^2+z^2)^3}{9}=3$ 

Đẳng thức xảy ra khi $x=y=z=1$

 

 

 

Bài 4:

Ch0 $a>0$ và $n$ là 1 số tự nhiên

Chứng minh rằng $a^n+\frac{1}{a^n}-2\geqslant n^2(a+\frac{1}{a}-2)$

Lời giải:

Bất đẳng thức tương đương với $(a^{n-1}+a^{n-2}+...+a+1)\geq n^2a^{n-1}$ (hiển nhiên theo AM-GM)

Cách khác:

Do tính đối xứng giữa a và $\frac{1}{a}$ nên ta có thể giả sử a ≥ 1.  đặt $\sqrt{a}$ =x ≥ 1.bdt $\Leftrightarrow$ $x^{2n}+\frac{1}{x^{2n}}-2 \geq n^{2}(x^{2}+\frac{1}{x^{2}}-2)\Leftrightarrow (x^{n}-\frac{1}{x^{n}})^{2}\geq n^{2}(x-\frac{1}{x})^{2} \Leftrightarrow $x^{n}-\frac{1}{x^{n}}\geq n(x-\frac{1}{x})$①.

Với x=1 thì ① đúng

Với x>1 thì ① $\Leftrightarrow x^{n-1} +x^{n-3} ...+\frac{1}{x^{n-3}}+\frac{1}{x^{n-1}}\geq n$ (đúng vì theo bđt AM-GM).

Dấu bằng xảy ra khi x=1 $\Leftrightarrow a=1$

 

 

 

Bài 5:

Cho $a,b,c,d$ là các số thực thỏa mãn $\left\{\begin{matrix} a+b+c+d=0\\a^2+b^2+c^2+d^2=2 \end{matrix}\right.$

Tìm GTLN của $P=abcd$

Lời giải:

Áp dụng AM-GM ta có 

$2=\sum a^{2}\geq 4\sqrt[4]{\prod a^{2}}\Rightarrow \sqrt{\left | abcd \right |}\leq \frac{1}{2}\Rightarrow abcd\leq \frac{1}{4}$

Dấu bằng xảy ra khi $a=b=-c=-d=\frac{1}{\sqrt{2}}$ và các hoán vị của chúng

 

 

 

Bài 6:

Cho $a,\,b,\,c\geq 0$ thỏa mãn $a+b+c=1.$ Tìm giá trị lớn nhất của biểu thức: $$P=abc\left(a^2+b^2+c^2\right)$$

Lời giải:

Ta có: $P=abc(a+b+c)(a^{2}+b^{2}+c^{2})\leq \frac{1}{3}(ab+bc+ca)^{2}(a^{2}+b^{2}+c^{2})$

 

Mặt khác, lại có: $(ab+bc+ca)^{2}(a^{2}+b^{2}+c^{2})\leq \left (\frac{(a+b+c)^{2}}{3} \right )^{3}=\frac{1}{27}$

 

Do đó: $P\leq \frac{1}{81}$

 

Dấu bằng xảy ra khi và chỉ khi $a=b=c= \frac{1}{3}$

 

 

 

Bài 7:

Cho các số thực $x,\,y>0$ thỏa mãn $3x+y\leq1.$ Tìm giá trị nhỏ nhất của biểu thức: $$S=\dfrac{1}{x}+\dfrac{1}{\sqrt{xy}}$$

Lời giải:

$S\geq \frac{1}{x}+\frac{1}{\sqrt{x(1-3x)}}$

$\geq \frac{1}{x}+\frac{2}{1-2x}=\frac{2}{x(1-x)}\geq \frac{8}{(x+1-x)^{2}}=8$

Dấu "=" xảy ra $\Leftrightarrow x=\frac{1}{4}$

 

 

 

Bài 8:

Cho các số thực a,b,c,x,y thỏa mãn $$ax-by=\sqrt{3}$$ .

Tìm GTNN của $F= a^{2}+b^{2}+x^{2}+y^{2}+ bx +ay$

Lời giải:

Sử dụng giả thiết $ax-by=\sqrt{3}$ ta có:
$$(a^2+b^2)(x^2+y^2)=(ax+by)^2+(ax-by)^2=(ax+by)^2+3$$
Áp dụng bất đẳng thức $Cauchy$ , suy ra:
$$a^2+b^2=x^2+y^2=(a^2+b^2)+(x^2+y^2) \\ \ge 2\sqrt{(a^2+b^2)(x^2+y^2)}=2\sqrt{(ax+by)^2+3}$$
Do đó, ta đưa về bài toán tìm GTNN của: $2\sqrt{x^2+3}+x$ trong đó $x=ax+by$
Ta có:
$$\left(2\sqrt{x^2+3}+x\right)^2=4(x^2+3)+4x\sqrt{x^2+3}+x^2 \\ = (x^2+3)+4x\sqrt{x^2+3}+4x^2+9 \\ = \left(\sqrt{x^2+3}+2x\right)^2+9\ge 9$$
$$\Rightarrow 2\sqrt{x^2+3}+x\ge 3$$
Vậy $\text{MinT}=\fbox{3}$
 
 
 

Bài 9:

Cho các số thực dương $a,b,c$. Tìm giá trị nhỏ nhất của biểu thức:

 

                                $P=\frac{2}{a+ \sqrt{ab}+ \sqrt[3]{abc}}-\frac{3}{\sqrt{a+b+c}}$

Lời giải:

$a+\sqrt{\frac{1}{2}a.2b}+\sqrt[3]{\frac{1}{4}a.b.4c}\leq a+\frac{1}{4}a+b+\frac{1}{12}a+\frac{1}{3}b+\frac{4}{3}c=\frac{4}{3}(a+b+c)$

Do đó $P\geq \frac{3}{2(a+b+c)}-\frac{3}{\sqrt{a+b+c}}$...

 

 

 

Bài 10:

Cho x,y là các số không âm thoả $x^{3}+y^{3}\leq 1$

 

Tìm giá trị lớn nhất của $P=2\sqrt{x}+\sqrt{y}$

Lời giải:

$(x^3+y^3)(\sqrt[5]{2^6}+1)^5\geqslant (2\sqrt{x}+\sqrt{y})^6$

$\Leftrightarrow 2\sqrt{x}+\sqrt{y}\leqslant \sqrt[6]{(\sqrt[5]{2^6}+1)^5}$

Vậy $Max(P)= \sqrt[6]{(\sqrt[5]{2^6}+1)^5}\Leftrightarrow \frac{a^3}{2\sqrt[5]{2}}=b^3=\frac{1}{2\sqrt[5]{2}+1}$

 

Chiều coppy tiếp, sau đó kiểm tra nội dung sau.

 

 

Bài 11:Cho các số a,b,c không âm không đồng thời bằng không. Chứng minh rằng;

$\sum \frac{2a^{2}-bc}{b^{2}-bc+c^{2}}\geq 3$

 

Lời giải:(vutuanhien)

 

Không mất tính tổng quát, ta có thể giả sử $b$ là số nằm giữa $a$ và $c$

BĐT đã cho tương đương với

$$\sum \frac{2a^2+(b-c)^2}{b^2-bc+c^2}\geq 6$$

Áp dụng BĐT Cauchy-Schwarz, ta có

$$\sum \frac{2a^2}{b^2-bc+c^2}\geq \frac{2(a^2+b^2+c^2)^2}{\sum a^2(b^2-bc+c^2)}=\frac{2(a^2+b^2+c^2)^2}{2\sum a^2b^2-abc\sum a}$$

$$\sum \frac{(b-c)^2}{b^2-bc+c^2}\geq \frac{[a(b-c)+b(a-c)+c(a-b)]^2}{2\sum a^2b^2-abc\sum a}=\frac{4b^2(a-c)^2}{2\sum a^2b^2-abc\sum a}$$

Do đó ta chỉ cần chứng minh

$$(a^2+b^2+c^2)^2+2b^2(a-c)^2\geq 6\sum a^2b^2-3abc\sum a (1)$$

Ta có 

$b^2(a-c)^2=[a(b-c)+c(a-b)]^2=a^2(b-c)^2+c^2(a-b)^2+2ac(a-b)(b-c)$

$\geq a^2(b-c)^2+c^2(a-b)^2$

Suy ra 

$$2b^2(a-c)^2\geq a^2(b-c)^2+b^2(c-a)^2+c^2(a-b)^2$$

$$\Rightarrow VT (1)\geq (\sum a^2)^2+2\sum a^2b^2-2abc\sum a$$

Do đó ta chỉ còn phải chứng minh 

$$(\sum a^2)^2+2\sum a^2b^2-2abc\sum a\geq 6\sum a^2b^2-3abc\sum a$$

$$\Leftrightarrow \sum a^4+abc\sum a\geq 2\sum a^2b^2$$

BĐT này hiển nhiên đúng theo BĐT Schur

$$\sum a^4+abc\sum a\geq \sum ab(a^2+b^2)$$

Và BĐT AM-GM

$$\sum ab(a^2+b^2)\geq 2\sum a^2b^2$$

Kết thúc chứng minh 

Đẳng thức xảy ra khi $a=b=c$ hoặc $a=b$, $c=0$ và các hoán vị.

 

Bài 12:(bosulan239)

Cho a,b,c là các số không âm không đồng thời bằng không.

CMR

$\frac{\sum a^{2}}{\sum ab}\geq \sum \frac{ab}{b^{2}+bc+c^{2}}$

Bài giải:(vutuanhien)

 

BĐT đã cho tương đương với

$\frac{a^2}{ab+bc+ca}-\frac{ab}{b^2+bc+c^2}+\frac{b^2}{ab+bc+ca}-\frac{bc}{c^2+ca+a^2}+\frac{c^2}{ab+bc+ca}-\frac{ca}{a^2+ab+b^2}\geq 0$

$\Leftrightarrow \sum \frac{ac(ac-b^2)}{b^2+bc+c^2}\geq 0$

Do $\frac{ac(ac-b^2)}{b^2+bc+c^2}=\frac{ac^2(a+b+c)}{b^2+bc+c^2}-ac$ nên BĐT đã cho có thể viết lại thành

$\sum \frac{ac^2(a+b+c)}{b^2+bc+c^2}\geq ab+bc+ca$

$\Leftrightarrow \sum \frac{ac^2}{b^2+bc+c^2}\geq \frac{ab+bc+ca}{a+b+c}$

Áp dụng BĐT Cauchy-Schwarz, ta có

$VT\geq \frac{(ab+bc+ca)^2}{\sum a(b^2+bc+c^2)}=\frac{ab+bc+ca}{a+b+c}$

Kết thúc chứng minh 

 

 


Bài 13: (nguyencuong123)

Cho a,b,c không âm thoả mãn: $a+b+c=3$

Chứng Minh: $\sum \frac{a+1}{ab+1}\geq 3$

 

 

Bài giải:(Juliel)

Áp dụng AM-GM cho vế trái, ta cần chứng minh :

$(a+1)(b+1)(c+1)\geq (ab+1)(bc+1)(ca+1)\Leftrightarrow abc+(ab+bc+ca)+(a+b+c)+1\geq a^{2}b^{2}c^{2}+abc(a+b+c)+(ab+bc+ca)+1\Leftrightarrow abc+4\geq a^{2}b^{2}c^{2}+3abc+1\Leftrightarrow a^{2}b^{2}c^{2}+2abc\leq 3$

Hiển nhiên đúng vì $abc\leq (\frac{a+b+c}{3})^{3}=1$

 


 


Bài 14:(Chrome98):Chứng minh bất đẳng thức sau với $a,b,c>0$ và $a+b+c=1$:

 

\[ \frac{a^2}{3a+1}+\frac{b^2}{3b+1}+\frac{c^2}{3c+1}\ge 24\left(\frac{a^2}{9a+1}+\frac{b^2}{9b+1}+\frac{c^2}{9c+1}\right)^2 \]

Bài giải: (Simpson Joe Donald)

 

 
 
$\bullet\ AM-GM:\ \dfrac{a^2}{9a+1}= \dfrac{a^2}{6a+(3a+1)}\le \dfrac{a^2}{2\sqrt{6a(3a+1)}}= \dfrac{a\sqrt{a}}{2\sqrt{6(3a+1)}}$ ;
$\bullet\ Cauchy-Schwarz:\ VP\le \left( \dfrac{a\sqrt{a}}{\sqrt{3a+1}}+  \dfrac{b\sqrt{b}}{\sqrt{3b+1}}+  \dfrac{c\sqrt{c}}{\sqrt{3c+1}}\right)^2\le (a+b+c).VT=VT$

 

Bài 15:(trauvang97:)Cho các số thực dương $a,b,c$ thoả mãn:

 

                             $\frac{2}{a^{2}+1}+\frac{2}{b^{2}+1}+\frac{2}{c^{2}+1}\geq 3$

 

Chứng minh rằng: $(a-2)^{2}+(b-2)^{2}+(c-2)^{2}\geq 3$

 

Bài giải:

 

(Nguyen Huy Tuyen)$\frac{2}{a^{2}+1}+\frac{2}{b^{2}+1}+\frac{2}{c^{2}+1}\geq 3\Leftrightarrow \sum \frac{(1-a)(1+a)}{a^2+1}\geqslant 0$

$(a-2)^{2}+(b-2)^{2}+(c-2)^{2}-3=\sum (a-3)(a-1)$

Ta có :$\sum (a-3)(a-1)-\sum \frac{2(1-a)(1+a)}{a^2+1}=\sum \frac{(a-1)^4}{a^2+1}\geqslant 0$

           $\Leftrightarrow \sum (a-3)(a-1)\geqslant \sum \frac{2(1-a)(1+a)}{a^2+1}\geqslant 0$

           $\Leftrightarrow (a-2)^{2}+(b-2)^{2}+(c-2)^{2}\geq 3$

 

 

Bài 16:(phanquockhanh)Cho $x,y,z >0 : xyz+x+z=y$ . Tìm giá trị lớn nhất của biểu thức : $P=\frac{2}{x^2+1} - \frac{2}{y^2+1} -\frac{4z}{\sqrt{z^2+1}}+\frac{3z}{(z^2+1).\sqrt{z^2+1}}$

(Trích đề thi thử số 2 – THTT)

Bài giải:

(trauvang97)Từ giả thiết ta có: $x=\frac{y-z}{1+yz}$.

 

Khi đó:

 

$P=\frac{2(1+yz)^{2}}{(y^{2}+1)(z^{2}+1)}-\frac{2}{y^{2}+1}-\frac{4z}{\sqrt{z^{2}+1}}+\frac{3z}{(z^{2}+1)\sqrt{z^{2}+1}}$

 

$P=\frac{2z(2y+(y^{2}-1)z)}{(y^{2}+1)(z^{2}+1)}-\frac{4z}{\sqrt{z^{2}+1}}+\frac{3z}{(z^{2}+1)\sqrt{z^{2}+1}}$

 

Do $\frac{2z(2y+(y^{2}-1)z)}{(y^{2}+1)(z^{2}+1)}=\frac{2z\sqrt{(2y+(y^{2}-1)z)^{2}}}{(y^{2}+1)(z^{2}+1)}\leq \frac{2z\sqrt{(4y^{2}+(y^{2}-1)^{2})(1+z^{2})}}{(y^{2}+1)(z^{2}+1)}=\frac{2z}{\sqrt{z^{2}+1}}$

 

Do đó:

 

$P\leq \frac{2z}{\sqrt{z^{2}+1}}-\frac{4z}{\sqrt{z^{2}+1}}+\frac{3z}{\sqrt{z^{2}+1}}\left ( 1-\frac{z^{2}}{z^{2}+1} \right )$

 

$P=-3t^{3}+t$ với $\frac{z}{\sqrt{z^{2}+1}}=t\in (0;1)$

 

Khảo sát hàm số trên ta thấy $maxP=\frac{2}{9}\Leftrightarrow x=\frac{\sqrt{2}}{2};y=\sqrt{2},z=\frac{\sqrt{2}}{4}$

 

 

 


Bài 17:(Toc Ngan)Cho $a,b,c >0$ và $a+b+c=3$

Chứng minh rằng : $8(\frac{1}{a}+\frac{1}{b}+\frac{1}{c})+9 \geq 10(a^2+b^2+c^2)$

 Bài giải:(babystudymaths)

 

Cách 1

Giải như sau:

Giả sử a là số lớn nhất trong 3 số a,b,c ,thế thì c nhỏ hơn 3 và không nhỏ hơn 1

ta thấy $9=(42a-48)+(42b-\frac{69}{2})+(42c-\frac{69}{2})$

Thay và BĐT ban đầu ta thấy tương đương

$(\frac{8}{b}-10b^{2}+42b-\frac{69}{2})+(\frac{8}{c}-10c^{2}+42c-\frac{69}{2})\geq 10a^{2}-\frac{8}{a}-42a+48\Leftrightarrow \frac{(16-5b)(2b-1)^{2}}{b}+\frac{(16-52)(2c-1)^{2}}{c}\geq \frac{4(5a-1)(a-2)^{2}}{a}$

Áp dụng BCS ,ta có:

VT $\geq \frac{(2b-1+2c-1)^{2}}{\frac{b}{16-5b}+\frac{c}{16-5c}}= \frac{4(a-2)^{2}}{\frac{b}{16-5b}+\frac{c}{16-5c}}$

Lúc này ta chỉ cần chứng minh 

$\frac{a}{5a-1}\geq \frac{b}{16-5b}+\frac{c}{16-5c}$

Mà $\frac{b}{16-5b}+\frac{c}{16-5c}\leq \frac{b}{16-5a}+\frac{c}{16-5a}= \frac{3-a}{16-5a}\leq \frac{a}{5a-1}\Leftrightarrow \frac{1}{(5a-1)(16-5a)}> 0$

ĐÚng theo giả thiết,từ đây ta suy ra đ.p.c.m

Đẳng thức xảy ra khi và chỉ khi a=b=1/2 ,c=2 cùng hoán vị

 

Cách 2: 

Ta có BĐT tương đương

f(abc,a+b+c,ab+bc+ca) =$8.\frac{ab+bc+ca}{abc}+9-10((a+b+c)^{2}-2(ab+bc+ca))\geq 0$

Nhận thấy đây là hàm đơn điệu trên R theo abc nên theo định lý ABC, hàm số đạt cực tiểu khi có 2 biến = nhau, nên a=b=$\frac{3-c}{2}$

Thay vào và chứng minh BĐT 1 biến c ,bài toán trở nên quá đơn giản 

 

 

 

 


Bài 18:(caybutbixanh)Cho $x;y;z> 0$.Chứng minh rằng :

$P=\frac{2xy}{(z+x)(z+y)}+\frac{2yz}{(x+y)(x+z)}+\frac{3xz}{(y+z)(y+x)}\geqslant \frac{5}{3}.$

 

(trích đề thi học sinh giỏi lớp 11-Quảng Bình 2011)

--------------------------------

 

 

 

(T M) Hướng giải:

 

Bằng khai triển trực tiếp ta đưa bất đẳng thức cần chứng minh thành

 

$$xy(x+y)+yz(y+z)+4xz\left ( x+z \right )\geq 10xyz$$

 

Điều này tương đương với

 

$$\frac{x+y}{z}+\frac{y+z}{x}+\frac{4(x+z)}{y}\geq 10$$

 

Áp dụng $AM-GM$ từng cặp là ra.

 

 


Bài 19:(supermath98)Cho các sô dương $a;b;c$ thỏa mãn $\large ab+ac+bc=3abc$. Tìm GTNN của biểu thức: 

 

$\large M=\frac{2\left ( a^{2}b^{2}+b^{2}c^{2}+a^{2}c^{2} \right )+abc}{a^{2}b^{2}c^{2}}$

 

 

Bài giải:(thanhdok14)

 

 

Vì $a, b, c>0$ nên từ điều kiện ban đầu, ta suy ra:

$\frac{1}{a}+\frac{1}{b}+\frac{1}{c}=3$

Đặt: $\left(\frac{1}{a};\frac{1}{b};\frac{1}{c}\right)\to (x;y;z)$

$\Rightarrow x+y+z=3$

$\Rightarrow xy+yz+zx\le 3$

Mặt khác: $M$ được viết lại thành:

$M=2\left(\frac{1}{a^2}+\frac{1}{b^2}+\frac{1}{c^2}\right)+\frac{1}{abc}$

$=2(x^2+y^2+z^2)+xyz$

Lại có: $x^2+y^2+z^2=9-2(xy+yz+zx)$

$xyz\ge \frac{(x+y+z)[4(xy+yz+zx)-(x+y+z)^2]}{9}=\frac{4(xy+yz+zx)-9}{3}$   (theo $schur$)

Từ đó ta có:

$M\ge \frac{4}{3}(xy+yz+zx)-4(xy+yz+zx)+15=\frac{-8}{3}(xy+yz+zx)+15\ge 7$   (vì $xy+yz+zx\le 3$)

Vậy $min M=7\Leftrightarrow a=b=c=1$

 

 


Bài 20:(duaconcuachua)

Cho $a,b,c$ là các số thực dương thỏa mãn $ab+bc+ca=abc$.

Chứng minh rằng $\frac{a^{4}+b^{4}}{ab(a^{3}+b^{3})}+\frac{b^{4}+c^{4}}{bc(b^{3}+c^{3})}+\frac{c^{4}+a^{4}}{ca(c^{3}+a^{3})}\geq 1$

 Bài giải:(Sagittius912)Theo bđt Chebyshev ta có

 

 

$\frac{a^4+b^4}{a^3+b^3}\ge \frac{a+b}{2}$

do đó

 

$\frac{a^{4}+b^{4}}{ab(a^{3}+b^{3})}+\frac{b^{4}+c^{4}}{bc(b^{3}+c^{3})}+\frac{c^{4}+a^{4}}{ca(c^{3}+a^{3})}\ge \frac{a+b}{2ab}+\frac{b+c}{2bc}+\frac{c+a}{2ca}=\frac{ab+bc+ca}{abc}=1$

 

Dấu đẳng thức xảy ra khi $a=b=c=3$

 

 


 

 

 

ở chỗ bài 6 làm sao mk ra dòng thứ 4 trên xuống thế ạ

 

 




#718506 Tìm m để $x^3-6x^2+9x-m=0$ có 3 nghiệm phân biệt

Đã gửi bởi LoveMath1234567 on 18-12-2018 - 17:54 trong Hàm số - Đạo hàm

Bài 1: Có 2 cách.

Cách 1: Dùng đồ thị biện luận số nghiệm của pt: $x^3 - 6x^2+ 9x = m$

Cách 2: dùng đạo hàm, tính 2 giá trị cực trị và cho chúng trái dấu. Tức y(CĐ).y(CT) < 0 .

Ta đều được đáp số 0< m < 4.

 

Bài 2: Đs 4 / $(e^x+e^{-x})^2$

bạn có thể trình bày cách 1 ra đc ko ạ




#718505 Giải PT bằng PP đặt 1 ẩn phụ

Đã gửi bởi LoveMath1234567 on 18-12-2018 - 17:48 trong Phương trình - Hệ phương trình - Bất phương trình

3 câu này đề giải bằng đặt 1 ẩn phụ nhưng giải bằng cách khác cũng được

attachicon.gifPTVT-Dat 1 an.png   

8,Đặt x: a, $ \sqrt{1-x^{2}}$: b 

hpt trở thành :

$ \left\{\begin{matrix} a^{3}+b^{3}= ab\sqrt{2} & & \\ a^{2}+b^{2}= 1& & \end{matrix}\right.$

ta có :$ \left ( a+b \right )^{2}= 1+2ab$

từ hpt ta có :

$\left ( a+b \right )\left ( a^{2}-ab+b^{2} \right )= ab\sqrt{2}$

$ \Leftrightarrow \left ( a+ b\right )^{2}\left ( 1-ab \right )^{2}-2a^{2}b^{2}= 0$

$ \Leftrightarrow \left ( 1+2ab \right )\left ( 1-ab \right )^{2}-2a^{2}b^{2}= 0$ ..Giải ra ab bằng 0,5  r quay lại hpt giải tiếp   :D




#718504 Chứng minh định lý lớn Fermat ngắn gọn bằng toán sơ cấp

Đã gửi bởi LoveMath1234567 on 18-12-2018 - 17:17 trong Nghiên cứu Toán học

Đây là cách chứng minh định lý lớn Fermat của tôi bằng toán sơ cấp, trong đó có cách cm đl trong trường hợp n = 3, n = 4 ngắn gọn và dễ hiểu. Hy vọng cách cm này làm thoả mãn với những ai quan tâm về việc cm đl lớn Fermat bằng toán sơ cấp. Rất mong mọi người quan tâm, thảo luận và góp ý.

cái này sử dụng cho dạng toán như nào ạ ....phạm vi ứng dụng có lớn ko ạ




#718478 Giải phương trình hệ phương trình bằng phương pháp lượng giác hóa

Đã gửi bởi LoveMath1234567 on 17-12-2018 - 19:08 trong Chuyên đề toán THPT

 

$\fbox{Ví dụ 7}$ Giải hệ phương trình:
 $\left\{\begin{matrix} x^2+4y^2=1 \\ 16x^5-20x^3+5x+512y^5-160y^3+10y+\sqrt{2}=0 \end{matrix} \right.$
Giải:
Rõ ràng từ phương trình tứ nhất của hệ ta thấy xuất hiện $A^2+B^2=1$ nên ta nghĩ ngay đến viện đặt $A=\sin t, B=\cos t$ khi đó chắc chắn sẽ tồn tại $ t \in (0;2\pi)$
Với $A=x,B=2y$ nên ta đặt $x=\sin t, y=\cos t, t \in(0;2\pi)$, ta được hệ phương trình:
$\left\{\begin{matrix} \sin^2 t+\cos^2 t=1 \\ 16\sin^5 t-20\sin^3 t +5 \sin t+16 \cos^5 t-20\cos^3 t +5 \cos t =-\sqrt{2} (*) \end{matrix} \right.$ 
Ta đi giải phương trình (*): Nhận thấy hệ số và bậc của hàm $\sin, \cos $ bằng nhau.Điều đó giúp ta liên tưởng đến công thức lượng giác
$ (*) \Leftrightarrow \sin 5t +\cos 5t =-\sqrt{2} \Leftrightarrow \sin 5t+\frac{\pi}{4}=-1 \Leftrightarrow t= \frac{-3\pi}{4}+k2\pi,k \in \mathbb{Z} $
Vì $ t \in (0;2\pi)$ mà $k \in \mathbb{Z}$ nên $ k=1;2;3;4;5 $ $ \Rightarrow t $ nhận các giá trị 
 $ t= \frac{\pi}{4}; \frac{13\pi}{20} ; \frac{21\pi}{20} ; \frac{29\pi}{20} ;\frac{27\pi}{20}$
Kết luận:Nghiệm hệ phương trình
$\left(\frac{\sqrt{2}}{2}; \frac{\sqrt{2}}{4} \right) ; \left(\sin \frac{13\pi}{20}; \frac{1}{2} \cos \frac{13\pi}{20} \right); \left( \sin \frac{21\pi}{20}; \frac{1}{2} \cos \frac{21\pi}{20} \right); \left(\sin \frac{29\pi}{20} ; \frac{1}{2} \cos \frac{29\pi}{20} \right); \left(\sin \frac{37\pi}{20}; \frac{1}{2} \cos \frac{37\pi}{20}\right) $  $\square$    
 
Nhận xét:
$\bullet$ Thoạt tiên, khi giải quyết hệ này ta thấy bậc ở phương trình thứ 2 rất lớn, lên tận bậc 5 $\rightarrow$ nghĩ đến việc sử dụng phương pháp hằng đẳng thức, phương pháp đánh giá , phương pháp hàm,..
$\bullet$ $x,y$ đứng độc lập và các hệ số các hạng tử cùng bậc bằng nhau nên ta nghĩ đến việc sử dụng phương pháp hàm để giải nhưng sự xuất hiện của $\sqrt{2}$ làm công việc trở nên khó khăn 
$\bullet$ Để ý kĩ một chút sự xuất hiện của phương trình thứ nhất $A^2+B^2=1$ và $\sqrt{2}$ đã làm cho ta liên tưởng đến phép đặt lượng giác quen thuộc được  nêu ở trên.
 
Đã liên tưởng đến phép đặt lượng giác nhưng công việc còn lại là khá rắc rối. Phương trình thứ 2 xuất hiện 3 loại bậc là 5,3,1 mà  công thức nhân 5 ẩn chứa chúng
Ghi nhớ:$\displaystyle \fbox{$\begin{matrix}  \cos 5 \alpha = 16 \cos^5 \alpha -20\cos^3 \alpha +5 \cos \alpha &  \\ \sin 5\alpha =16 \sin^5 \alpha -20 \sin^3 \alpha +5 \sin \alpha & \end{matrix}$ } $
Bài tập tương tự: Giải hệ phương trình:
$\left\{\begin{matrix} (2x+3y)^2=1+12xy \\ 512x^5-160x^3+12x+3888y^5-540y^3+18y=0 \end{matrix} \right.$
$\fbox{Ví dụ 8}$ Giải hệ phương trình:
 $\left\{\begin{matrix} 3\left(x+\frac{1}{x}\right)=4\left(y+\frac{1}{y}\right)=5\left(z+\frac{1}{z} \right) \:\: (1)\\ xy +yz +zx=1 \hspace{4.1cm}(2) \end{matrix} \right.$ 
Giải: 
Điều kiện $xyz \neq 0$ từ $xy+yz+zx=1$ suy ra $x,y,z$ phải cùng dấu
Nhận thấy nếu $(x;y;z) $ là một nghiệm của hệ thì $(-x;-y;-z) $ cũng là nghiệm của hệ . Do vậy ta chỉ cần tìm nghiệm dương của hệ $\rightarrow$ nghiệm còn lại
Xét trường hợp $ x,y,z > 0$ 
Vì có sự xuất hiện $xy+yz+zx=1$ nên ta đặt $x=\tan \alpha ; y=\tan \beta ; z= \tan \gamma \left (0< \alpha,\beta,\gamma <\frac{\pi}{2} \right)$
Từ phương trình (2): $\tan \alpha.\tan \beta+ \tan \beta.\tan \gamma+ \tan \gamma.\tan \alpha=1$
 $\Leftrightarrow \tan \beta(\tan \alpha+\tan \gamma)=1-\tan \gamma \tan \alpha$
$\Leftrightarrow \tan \beta = \frac{1- \tan \gamma \tan \alpha}{ \tan \alpha+ \tan \gamma}= \cot(\alpha+\gamma)$
$\Leftrightarrow \alpha+ \beta+\gamma = \frac{\pi}{2}$
Từ phương trình (1): $3 \frac{\tan^2 \alpha}{\tan \alpha} =4 \frac{\tan^2 \beta+1}{\tan \beta}=5 \frac{\tan^2 \gamma+1}{\tan \gamma}$
$\Leftrightarrow \frac{3}{\sin 2\alpha}= \frac{4}{\sin 2\beta} = \frac{5}{\sin 2\gamma}$ 
 Ta có hệ tương đương: $\left\{\begin{matrix} \frac{3}{\sin 2\alpha}= \frac{4}{\sin 2\beta} = \frac{5}{\sin 2\gamma} \\ \\ 0< \alpha,\beta,\gamma < \frac{\pi}{2} ; \alpha+\beta+\gamma =\frac{\pi}{2} \end{matrix} \right.$ 
Từ hệ trên suy ra $ 2\alpha; 2\beta ; 2\gamma $ là các góc của tam giác có cạnh tương ứng là 3;4;5 mà 3;4;5 là bộ 3 PY-TA-GO
Theo định lý sin trong tam giác $\rightarrow 2\gamma =90^\circ  \Rightarrow \gamma =45^\circ \Rightarrow z= \tan 45^\circ  =1$ 
$\tan 2\alpha= \frac{2\tan \alpha}{1-\tan^2 \alpha} =\frac{3}{4} \Rightarrow \tan \alpha = \frac{1}{3}=x$
$\tan 2\beta= \frac{2\tan \beta}{1-tan^2 \beta}= \frac{4}{3} \Rightarrow \tan \beta=\frac{1}{2}=y$
Vậy hệ có 2 nghiệm là $\left(\frac{1}{3};\frac{1}{2};1 \right) ; \left(\frac{-1}{3} ; \frac{-1}{2};-1 \right) $     $\square$ 
$\fbox{Ví dụ 9}$ Giải hệ phương trình:
 $\left\{\begin{matrix} x+y+z=1 \hspace{3.9cm}(1)\\ \frac{x}{x+yz} +\frac{y}{y+zx}+\frac{z}{z+xy} = \frac{9}{4} \:\:\:\:\:\:\:(2) \end{matrix} \right.$
Giải:
Nhận thấy $x,y,z=0$ không phải là nghiệm hệ
Viết lại phương trình (1) dưới dạng $\sqrt{\frac{xy}{z}}\sqrt{\frac{xz}{y}}+\sqrt{\frac{yz}{x}}\sqrt{\frac{yx}{z}}+\sqrt{\frac{zx}{y}}\sqrt{\frac{zy}{x}}=1$
Đặt $\sqrt{\frac{xy}{z}}= \tan \frac{A}{2} , \sqrt{\frac{xz}{y}}=\tan \frac{B}{2}, \sqrt{\frac{yz}{x}}=\tan \frac{C}{2}; A,B,C \in(0,\pi)$
ta được $\tan{\frac{A}{2}} \tan{\frac{B}{2}} + \tan{\frac{B}{2}} \tan{\frac{C}{2}}+\tan{\frac{C}{2}} \tan{\frac{A}{2}}=1$
Tương tự như ví dụ trên dễ dàng suy ra $A+B+C= \pi$
Phương trình (2):$\frac{x}{x+yz}+\frac{y}{y+zx}+\frac{z}{z+xy} =\displaystyle \frac{1}{1+tan^2\frac{A}{2}}+\frac{1}{1+tan^2\frac{B}{2}}+\frac{1}{1+tan^2\frac{C}{2}}= \frac{9}{4}$
$\Leftrightarrow \cos^2 \frac{A}{2}+\cos^2 \frac{B}{2}+\cos^2 \frac{C}{2}=\frac{9}{4}$
$\Leftrightarrow \frac{3+\cos A+\cos B+\cos C}{2}=\frac{9}{4}$
$\Leftrightarrow \cos A+ \cos B+\cos C= \frac{3}{2}$ 
$\Leftrightarrow 1-2\sin^2 \frac{A}{2} +2 \cos \frac{B+C}{2} \cos \frac{B-C}{2}= \frac{3}{2}$ 
$\Leftrightarrow 4\sin^2 \frac{A}{2} +2 \sin \frac{A}{2} \cos \frac{B-C}{2}=\frac{3}{2}$ (*) 
$\triangle  ' =4(\cos^2 \frac{B-C}{2}-1) \geqslant 0 $ .Mặt khác $\cos^2 \frac{B-C}{2}-1 \leqslant 0$ 
 Nên (3) $\Leftrightarrow \left\{\begin{matrix} 2\sin \frac{A}{2}=\cos \frac{B-C}{2} \\ \sin \frac{B-C}{2}=0 \end{matrix} \right.$ $\Leftrightarrow A=B=C=\frac{\pi}{3}$ .
Từ đó suy ra $x=y=z=\frac{1}{3} \square$
$\fbox{Ví dụ 10}$:Tìm tất cả các số thực $x,y,z$ thỏa mãn:
 $x^6+y^6+z^6-6(x^4+y^4+z^4)+10(x^2+y^2+z^2)-2(x^3y+y^3z+z^3x)+6(xy+yz+zx)=0$
Giải:  Phương trình tương đương với 
$(x^3-3x-y)^2+(y^3-3y-z)^2+(z^3-3z-x)^3=0 \Leftrightarrow \left\{\begin{matrix} y=x^3-3x \\ x=z^3-3z \\ z=y^3-3y \end{matrix} \right.(I)$
+) Nếu $x>2$ thì $y=x^3-3x=x(x^2-3)>2 \Rightarrow z=y(y^2-3)>2$.Ta cộng 3 vế hệ $(I)$ ta được:
$0=x^3+y^3+z^3-4x-4y-4z=x(x^2-4)+y(y^2-4)+z(z^2-4)>0$ (Vô lý)
+) Tương tự với trường hợp $x<2$ thì hệ (I) không có nghiệm.Vậy $\left |x\right | \leqslant 2$
Với điều kiện đó ta đặt $x=2\cos t , t \in [0;\pi] $ ta đươc hệ: $\left\{\begin{matrix} y=2(4\cos^3 t -3\cos t)=2 \cos 3t \\ x=2(4\cos^3 3t- 3\cos 3t)= 2\cos 9t \\ z= 2(4\cos^3 9t -3\cos 9t)=2\cos 27t \end{matrix} \right.$
Từ hệ trên suy ra $\cos t= \cos 27t \Leftrightarrow t= k\frac{\pi}{13},k \in \mathbb{Z}$ hoặc $t= l\frac{\pi}{14}, l \in \mathbb{Z}$
mà $t \in[0;\pi]$ nên $k=0;1;2;...;13$ hoặc $l=0;1;2;..;14$
Vậy bộ 3 số $(x,y,z)$ cần tìm là $(2\cos t; 2\cos 3t; 2\cos 9t)$ với  $t=k\frac{\pi}{13},k=0;1;2;...;13$ hoặc $t=l\frac{\pi}{14},l=0;1;2;..;14$.Có 27 bộ 3 số thỏa mãn $\square$
Nhận xét:
Không giống như các ví dụ trước,điều kiện của biến thường được thấy rõ từ điều kiện xác định của phương trình.Ở ví dụ này,chúng ta phải tìm điều kiện chặt của biến để từ đó tìm ra phép đặt lượng giác.
Bài tập tương tự: Tìm tất cả các giá trị của tổng $S=x+y+z$;biết rằng $x,y,z$ là nghiệm hệ phương trình:
$\left\{\begin{matrix} x=y(4-y) \\ y=z(4-z) \\ z=x(4-x) \end{matrix} \right.$ 
III.Bài tập tự luyện
Giải các phương trình và hệ phương trình sau 
$\fbox{1}$ $\sqrt{1-x}=2x^2-1+2x\sqrt{1-x^2}$
$\fbox{2}$ $2x+(4x^2-1)\sqrt{1-x^2}=4x^3+\sqrt{1-x^2}$
$\fbox{3}$ $2-\frac{x}{\sqrt{1-x^2}}=2x^2$ 
$\fbox{4}$ $8x.(2x^2-1)(8x^4-8x^2+1)=1, x \in (0;1) $
$\fbox{5}$ $\left\{\begin{matrix} x^2+y^2+z^2=1 \\ 2xy+yz+zx=\frac{1+\sqrt{3}}{2} \end{matrix} \right.$
$\fbox{6}$ $\left\{\begin{matrix} x+y+z=xyz \\ x(y^2-1)(z^2-1)+y(x^2-1)(z^2-1)+z(x^2-1)(y^2-1)=0 \end{matrix} \right.$
$\fbox{7}$ $\left\{\begin{matrix} (1+x^2+x^2y+y)^2=8(x^2+x^2y) \\ (1+y^2+y^2z+z)^2=8(y^2+y^2z) \\ (1+z^2+z^2x+x)^2=8(z^2+z^2x) \end{matrix} \right.$
$\fbox{8}$ $\left\{\begin{matrix} x+y+z=1 \\ \sqrt{\frac{xy}{z+xy}} +\sqrt{\frac{yz}{x+yz}}\sqrt{\frac{zx}{y+zx}} \end{matrix} \right.$ 
$\fbox{9}$ $ \left\{\begin{matrix} 0<x,y,z<1 \\ xy+yz+zx=1 \\ \frac{x}{1-x^2}+\frac{y}{1-y^2} +\frac{z}{1-z^2} = \frac{3\sqrt{3}}{2} \end{matrix} \right.$ 

$\fbox{10}$  $\left\{\begin{matrix} z^2+2xyz=1 \\ 3x^2y^2+3xy^2=1+x^3y^4 \\ z+zy^4+4y^3=4y+6y^2z \end{matrix} \right.$

$\fbox{11}$ $\left\{\begin{matrix} 2z(x+y)+1=x^2-y^2 \\ y^2+z^2=1+2xy+2zx-2yz \\ y(3x^2-1)=-2x(x^2+1) \end{matrix} \right.$ 

$\fbox{12}$Tìm nghiệm dương của hệ:$\left\{\begin{matrix} x+y+z=a+b+c\\ 4xyz-a^2x-b^2y-c^2z=abc \end{matrix} \right.$ trong đó $a,b,c$ là các số dương cho trước

 

Vì kiến thức còn hạn hẹp nên mong các thành viên VMF đóng góp nhiều bài toán hay về phương pháp để cho bài  viết được hoàn chỉnh.Hãy cùng thảo luận tại http://diendantoanho...c-hóa/?p=458492

 

 

Ứng dụng của pp này có rộng ko ạ




#718473 xin tiền thế nào?

Đã gửi bởi LoveMath1234567 on 17-12-2018 - 18:38 trong Toán học lý thú

Để ợt Alph@ giai cho
9567+1085=10652
S=9 ; M=1 ;O=0 ;E=6 ;N=6 ; D=7; R=8 ;Y=2
Xong rùi nhé!
Đố bạn:
SEND
+ HEL LO
HAPPY
giống của bạn hông !
Giải thử đi

bạn làm sao mk ra z bn




#718468 Tìm m để pt có ít nhất 1 nghiệm lớn hơn hoặc bằng 1

Đã gửi bởi LoveMath1234567 on 17-12-2018 - 17:37 trong Toán học lý thú

Cảm ơn bạn, mình đã sửa lại ở trên.

bạn cho mik hỏi làm sao mk bn ra kết quả như dòng thứ 2 dưới lên vậy ạ .....Bạn phân tích ra cho mik vs




#718463 Tìm m để pt có ít nhất 1 nghiệm lớn hơn hoặc bằng 1

Đã gửi bởi LoveMath1234567 on 17-12-2018 - 13:22 trong Toán học lý thú

Ta xét mệnh đề đảo của mệnh đề đã cho : Tìm $m$ sao cho phương trình đã cho vô nghiệm hoặc nếu có nghiệm thì các nghiệm đều bé hơn $1$.

Xét $\Delta'=(m+1)^2-2(m^2+4m+3)=-m^2-6m-5=-(m+2)(m+3)$.

Trường hợp 1: Phương trình vô nghiệm

Ta có phương trình đã cho vô nghiệm khi và chỉ khi $\Delta'<0\iff -(m+2)(m+3)<0\iff m>-2\text{ hoặc }m<-3$.

Trường hợp 2: Phương trình đã cho có nghiệm và các nghiệm đều bé hơn $1$.

Giả sử hai nghiệm đó là $x_1,x_2$ ($x_1,x_2$ không nhất thiết phân biệt)

Ta có: $\Delta\ge 0\iff -(m+2)(m+3)\ge 0\iff -3\le m\le -2(2)$.

Và $\left\{\begin{array}{I} x_1+x_2<2\\(x_1-1)(x_2-1)>0 \end{array}\right.$

$\iff \left\{\begin{array}{I} x_1+x_2<2\\x_1x_2-(x_1+x_2)+1>0 \end{array}\right.(1)$

Theo định lí Vi-et ta có:

$(1)\iff \left\{\begin{array}{I} -m-1<2\\\frac{m^2+4m+3}{2}+m+1+1>0 \end{array}\right.$

$\iff \left\{\begin{array}{I} m>-3\\\frac{m^2+6m+7}{2}>0 \end{array}\right.$

$\iff \left\{\begin{array}{I} m>-3\\ m<-3-\sqrt{2}\text{ hoặc }m>-3+\sqrt{2} \end{array}\right.(3)$

Từ $(2)(3)$ ta thấy không có giá trị nào của $m$ thỏa mãn.

Vậy từ 2 trường hợp trên ta thấy mệnh đề đảo xảy ra khi $m>-2\text{ hoặc }m<-3$.

Quay lại bài toán. Ta kết luận, các giá trị của $m$ cần tìm là $-3\le m\le -2$.

Hình như bạn phân tích thành nhân tử sai r pải ko ạ ....$ -m^{2}-6m-5= -\left ( x+1 \right )\left ( x+5 \right )$  




#718446 Giải phương trình

Đã gửi bởi LoveMath1234567 on 16-12-2018 - 21:25 trong Phương trình, hệ phương trình và bất phương trình

Thấy pt có 1 nghiệm vô tỉ. Mình có học cái công thức nên áp dụng vào thôi. 

Công thức như nào z ạ ...chị có thể chia sẻ đc ko ạ ...Cám ơn chị




#718445 Tìm m để pt có ít nhất 1 nghiệm lớn hơn hoặc bằng 1

Đã gửi bởi LoveMath1234567 on 16-12-2018 - 21:20 trong Toán học lý thú

Cho $ f\left ( x \right )= 2x^{2}+2\left ( m+1\right )x+m^{2}+4m+3$ 

a, Tìm m để $ f\left ( x \right )= 0$ có ít nhất 1 nghiệm lớn hơn hoặc bằng 1